Đến nội dung

nhatquangsin

nhatquangsin

Đăng ký: 14-01-2013
Offline Đăng nhập: 23-07-2018 - 20:04
***--

#466393 $AB,ML,NK$ đồng quy

Gửi bởi nhatquangsin trong 24-11-2013 - 00:50

Xét bài toán sau:

Cho đường tròn tâm $O$ và tứ giác $ABCD$ nội tiếp đường tròn. $AB$ cắt $CD$ tại $E$, $AC$ cắt $BD$ tại $G$. $EG$ cắt $(O)$ tại hai điểm $M,N$. Khi đó $(EGMN)=-1$.

Chứng minh khá đơn giản như sau

Vẽ $AD$ cắt $BC$ tại $F$. Từ $F$ kẻ hai tiếp tuyến $FP,FQ$ tới $(O)$. $PQ$ cắt $BC,AD$ tại $K,L$. Theo tính chất thì $(ELCB)=(EKDA)=-1$ nên suy ra $AD,PQ,BC$ đồng quy tại $E$.Do đó dễ dàng suy ra $(EGMN)=-1$

-------------------

Theo bài toán trên thì $(DBLN)=(DAMK)=-1$ nên suy ra $AB,ML,NK$ đồng quy(trong đó $D$ là trực tâm tam giác $ABC$)

Hình gửi kèm

  • Snap108.png



#465517 $ (a+b-c-1)(b+c-a-1)(c+a-b-1) \le 8 $

Gửi bởi nhatquangsin trong 20-11-2013 - 17:50

Bài 21: Giả sử a,b,c là ba cạnh của một tam giác .cmr $\sum \frac{a}{\sqrt{a^{2}+3bc}}\geq \frac{3}{2}$

 

Do bất đẳng thức thuần nhất bậc 0 nên ta chuẩn hoá $a+b+c=3$

Ta có $\sum _{cyc}\frac{a}{\sqrt{a^2+3bc}}\geq \frac{9}{\sum a\sqrt{a^2+3bc}}\geq \frac{36}{\sum 5a^2+3bc}=\frac{36}{5(a+b+c)^2-7(ab+bc+ca)}\geq \frac{3}{2}$

(Do $ab+bc+ca\leq3$)

sorry các bạn, cách giải này có 1 lỗi

----------------------------------------

Bài 22: Cho $a_1,a_2,...,a_n$ là các số dương. Chứng minh rằng

$(1+\frac{a_1^2}{a_2})(1+\frac{a_2^2}{a_3})...(1+\frac{a_n^2}{a_1})\leq (1+\frac{a_1^3}{a_2a_3})(1+\frac{a_2^3}{a_3a_4})...(1+\frac{a_n^3}{a_1a_2})$


  • LNH yêu thích


#464558 $MA,PB,d$ đồng quy

Gửi bởi nhatquangsin trong 15-11-2013 - 21:16

Gửi hêm cái hình

Hình gửi kèm

  • 123456.PNG



#464301 $MA,PB,d$ đồng quy

Gửi bởi nhatquangsin trong 14-11-2013 - 16:14

Cho hai đường tròn $(O)$ và $(O')$ có $d$ là trục đẳng phương. Lấy điểm $I$ bất kì trên $d$. Từ $I$ kẻ hai tiếp tuyến $IA$ và $IB$ lần lượt đến $(O)$ và $(O')$. $IA,IB$ cắt $OO'$ tại $C,D$. Lấy $P$ bất kì trên $d$. $PC$ cắt $(O)$ tại $M,N$ theo thứ tự. Tương tự $PD$ cắt $(O')$ tại $Q,R$ theo thứ tự. Chứng minh rằng $MA,QB,d$ đồng quy.




#462937 $a^2b+b^2c+c^2a\leq \frac{4}{27}$

Gửi bởi nhatquangsin trong 08-11-2013 - 20:36

Cho $a,b,c$ là các số không âm và có tổng bằng $1$. Chứng minh $a^2b+b^2c+c^2a\leq \frac{4}{27}$


  • LNH yêu thích


#454800 Đề thi chọn đội tuyển học sinh giỏi trường THPT chuyên Lê Quý Đôn Bình Định,...

Gửi bởi nhatquangsin trong 03-10-2013 - 01:10



 

ĐỀ THI CHỌN ĐỘI TUYỂN HỌC SINH GIỎI LỚP 12 TRƯỜNG THPT CHUYÊN LÊ QUÝ ĐÔN 

BÌNH ĐỊNH- NĂM 2013-2014

________________________________

 

 

 

Bài 2: Cho ba số thực $a,b,c$ biến thiên trong đoạn $[1;2]$ và thỏa mãn: $a+b+c=4$

Tìm giá trị lớn nhất của biểu thức:$$P=\frac{a}{b+c}+\frac{b}{c+a}+\frac{c}{a+b}$$

Ta có $P+3=(a+b+c)\left ( \frac{1}{a+b}+\frac{1}{b+c}+\frac{1}{c+a}\right)=4\left ( \frac{1}{a+b}+\frac{1}{b+c}+\frac{1}{c+a} \right)$

Đặt $x=a+b,y=b+c,z=c+a$, ta có $x,y,z\in \left [ 2,4 \right ]$ và $x+y+z=8$. ta đi chứng minh $\frac{1}{x}+\frac{1}{y}+\frac{1}{z}\leq \frac{7}{6}$. Khi đó $minP=\frac{5}{3}$.

Đầu tiên ta có các bất đẳng thức sau

$x\geq 2,x+y=4+b\geq 5$

Sử dụng khai triển Abel ta có

$\frac{7}{6}=\frac{1}{3}+\frac{1}{3}+\frac{1}{2}=\frac{1}{x}.\frac{x}{3}+\frac{1}{y}.\frac{y}{3}+\frac{1}{z}.\frac{z}{2}$

$=\left ( \frac{1}{x}-\frac{1}{y} \right ).\frac{x}{3}+\left ( \frac{1}{y}-\frac{1}{z} \right ).\left ( \frac{x}{3}+\frac{y}{3} \right )+\frac{1}{z}.\left ( \frac{x}{3}+\frac{y}{3}+\frac{z}{2} \right )\geq$

$\geq \frac{2}{3}\left ( \frac{1}{x}-\frac{1}{y} \right )+\frac{5}{3}\left ( \frac{1}{y}-\frac{1}{z} \right )+\frac{1}{z}\left ( \frac{8}{3}+\frac{z}{6} \right )$

$=\frac{2}{3}.\frac{1}{x}+\frac{1}{6}+\frac{1}{y}+\frac{1}{z}\geq \frac{1}{x}+\frac{1}{y}+\frac{1}{z}$ (đpcm)

Vậy $minP=\frac{5}{3}$ khi $b=c=1,a=2$ và các hoán vị




#454158 $W’_{B}, W’_{C}$ chia đôi chu vi tâm giác ABC

Gửi bởi nhatquangsin trong 29-09-2013 - 23:44

Cho tam giác ABC, $W_{B},W_{C}$ là các đường tròn bàng tiếp các góc B,C. $W’_{B}, W’_{C}$ lần lượt đối xứng với $W_{B},W_{C}$ qua trung điểm của AC,AB. Chứng minh rằng trục đẳng phương của $W’_{B}, W’_{C}$ chia đôi chu vi tâm giác ABC




#452659 .tính góc BDC

Gửi bởi nhatquangsin trong 23-09-2013 - 21:57

cho tam giác ABC cân tại A có góc A =$20^{\circ}$ trên AB lấy điểm D sao cho BC=AD.tính góc BDC

Dựng tam giác đều ABM trên bờ AB chứa C..................=> BDC=30




#452256 Topic ôn thi HSG lớp 10 Đồng Bằng Bắc Bộ và Olympic 30-4

Gửi bởi nhatquangsin trong 22-09-2013 - 09:54

Vậy để mình post bài lên vậy  :biggrin:

$\boxed{7}$ Cho các số thực $a,b,c,d$ thỏa mãn $\left\{\begin{matrix} 0<a\leq b\leq c\leq d\\ \dfrac{1}{a}+\dfrac{2}{b}+\dfrac{d}{c}\geq 3\\ \dfrac{2}{b}+\dfrac{d}{c}\geq 2\\ \end{matrix}\right.$

Chứng minh rằng $a^{k}+b^{k}+c^{k}-d^{k}\leq 2^{k}+1$ với $k\in \mathbb{Z}^{+}$

(Tổng quát đề nghị thi Olympic 30-4 THPT Chuyên Lê Qúy Đôn, Bình Định)

Ta thiết lập các bđt sau(sử dụng Cauchy-Schwarz)

$\left ( \frac{d}{c} \right )^{k}\geq 1,\left ( \frac{d}{c} \right )^{k}+\left ( \frac{2}{b} \right )^{k}\geq 2,\left ( \frac{d}{c} \right )^{k}+\left ( \frac{2}{b} \right )^{k}+\left ( \frac{1}{a} \right )^{k}\geq 3$.

sử dụng khai triển Abel ta có

$d^{k}+2^{k}+1=\left ( c^{k}-b^{k} \right ).\left ( \frac{d}{c} \right )^{k}+\left ( b^{k}-a^{k} \right )\left ( \left ( \frac{d}{c} \right )^{k}+\left ( \frac{2}{b} \right )^{k} \right )+a^{k}\left ( \left ( \frac{d}{c} \right )^{k}+\left ( \frac{2}{b} \right )^{k}+\left ( \frac{1}{a} \right )^{k} \right )$

$\geq c^{k}-b^{k}+2(b^{k}-a^{k})+3a^{k}=a^{k}+b^{k}+c^{k}$

dấu = tại $a=1,b=2,c=d$




#452244 Topic ôn thi HSG lớp 10 Đồng Bằng Bắc Bộ và Olympic 30-4

Gửi bởi nhatquangsin trong 22-09-2013 - 09:31

Không ai gửi bài cả :huh:

 

$\boxed{6}$ Cho $\Delta ABC$. Chứng minh rằng $\frac{1}{R}+\frac{1}{r}\geq \frac{9\sqrt{3}}{2p}$

đặt $x=p-a,y=p-b,z=p-c$

bất đẳng thức cần chứng minh tương đương

$\frac{4\sqrt{(x+y+z)^3.xyz}}{(x+y)(y+z)(z+x)}+\sqrt{\frac{(x+y+z)^3}{xyz}}\geq \frac{9\sqrt{3}}{2}$.

Ta có

$VT\geq \frac{27}{2}.\sqrt{\frac{xyz}{(x+y+z)^3}}+\sqrt{\frac{(x+y+z)^3}{xyz}}\geq 3\sqrt{3}+\frac{3\sqrt{3}}{2}=\frac{9\sqrt{3}}{2}$




#452160 Tô màu: Chứng minh có đúng 33 ô đen

Gửi bởi nhatquangsin trong 21-09-2013 - 22:10

Số ô 2x3 và 3x2 của bảng là $10.4+9.5=95$ ô, do đó số ô đen tính qua các ô 2x3 và 3x2 là $95.2=190$(sẽ có ô thể trùng nhau) và số ô của bảng tính qua 95 ô 2x3 và 3x2 là $95.6=570$(ô trùng nhau). Gọi $a$ là số ô đen của bảng thì $\frac{a}{99}=\frac{190}{570}\rightarrow a=33$




#451281 $ (a+b-c-1)(b+c-a-1)(c+a-b-1) \le 8 $

Gửi bởi nhatquangsin trong 17-09-2013 - 20:19

Bài 6

    Cho các số dương a,b,c thỏa mãn $abc=1$.CMR

                    $\frac{1}{a+\sqrt{3a+1}}+\frac{1}{b+\sqrt{3b+1}}+\frac{1}{c+\sqrt{3c+1}}\leq 1$

bài 6 là dấu lớn hơn hoặc bằng chứ nhỉ




#448782 $ (a+b-c-1)(b+c-a-1)(c+a-b-1) \le 8 $

Gửi bởi nhatquangsin trong 08-09-2013 - 11:07

Bài 5

Cho $a,b,c$ là các số thực dương. Chứng minh rằng:

$\sqrt{a^{2}-ab+b^{2}}+\sqrt{b^{2}-bc+c^{2}}\geq \sqrt{a^{2}+ac+c^{2}}$


  • LNH yêu thích


#448656 $ (a+b-c-1)(b+c-a-1)(c+a-b-1) \le 8 $

Gửi bởi nhatquangsin trong 07-09-2013 - 23:58

Bài 4

Cho $a,b,c$ là các số nguyên dương có tổng là $3$. Chứng minh rằng:

$18\sum_{cyc}\frac{1}{(3-c)(4-c)}+2(ab+bc+ca)\geq 15$


  • LNH yêu thích


#448355 $ac+bd=(b+d+a-c)(b+d-a+c)$

Gửi bởi nhatquangsin trong 06-09-2013 - 23:58

Cho các số nguyên dương $a,b,c,d$ với $a>b>c>d>0$. Giả sử $ac+bd=(b+d+a-c)(b+d-a+c)$. Chứng minh rằng $ab+cd$ là hợp số